Find values of $a$ for which the function is periodic.












1












$begingroup$


Given a positive integer $m$ consider the sequence ${a_n}$ of positive integers defined by the initial term $a_0=a$ and the recurrence relation



$a_{n+1} = a_n/2$ if $a_n$ is even;
$a_n+m$, if $a_n$ is odd.



Find all values of $a$ for which the sequence is periodic.



I tried some kind of case-checking kind of solution, but that did not lead me anywhere. Please help.










share|cite|improve this question











$endgroup$

















    1












    $begingroup$


    Given a positive integer $m$ consider the sequence ${a_n}$ of positive integers defined by the initial term $a_0=a$ and the recurrence relation



    $a_{n+1} = a_n/2$ if $a_n$ is even;
    $a_n+m$, if $a_n$ is odd.



    Find all values of $a$ for which the sequence is periodic.



    I tried some kind of case-checking kind of solution, but that did not lead me anywhere. Please help.










    share|cite|improve this question











    $endgroup$















      1












      1








      1





      $begingroup$


      Given a positive integer $m$ consider the sequence ${a_n}$ of positive integers defined by the initial term $a_0=a$ and the recurrence relation



      $a_{n+1} = a_n/2$ if $a_n$ is even;
      $a_n+m$, if $a_n$ is odd.



      Find all values of $a$ for which the sequence is periodic.



      I tried some kind of case-checking kind of solution, but that did not lead me anywhere. Please help.










      share|cite|improve this question











      $endgroup$




      Given a positive integer $m$ consider the sequence ${a_n}$ of positive integers defined by the initial term $a_0=a$ and the recurrence relation



      $a_{n+1} = a_n/2$ if $a_n$ is even;
      $a_n+m$, if $a_n$ is odd.



      Find all values of $a$ for which the sequence is periodic.



      I tried some kind of case-checking kind of solution, but that did not lead me anywhere. Please help.







      number-theory recurrence-relations






      share|cite|improve this question















      share|cite|improve this question













      share|cite|improve this question




      share|cite|improve this question








      edited Jan 12 at 16:47









      SinTan1729

      2,682723




      2,682723










      asked Jan 12 at 16:44









      YellowYellow

      16011




      16011






















          1 Answer
          1






          active

          oldest

          votes


















          0












          $begingroup$


          Fact 1: If there are two distinct $k_1$ and $k_2$ s.t. $a_{k_1}=a_{k_2}$ then the sequence repeats.




          If $m$ is odd (the crux of the problem) then each $a_k$ is no larger than $a_1 + 3m$ [make sure you see why or check the proof below; essentially you get to add by $m$ only once before you have to divide by 2]. As each $a_k$ is positive it follows that $a_k in {1,2,ldots, a_1+3m}$ for all $k$. Therefore it follows from Fact 1 that the sequence repeats.



          If $m$ is even and positive then let $i_0$ be the smallest integer such that $a_12^{-i_0}$ is no longer even--or equivalently $2^{-i_0}a_1$ is odd. Then $a_2 = a_12^{-1}$, $ldots ,$ $a_{i_0+1} = 2^{-i_0}a_1$, while $a_{i_0+1+j} = a_12^{-i_0} + jm$ for each nonnegative integer $j$ [make sure you see why; note that $a_{i_0+j+1} = a_12^{-i_0} + jm$ is an odd integer as $a_12^{-i_0}$ is odd and $m$ is even]. So the function is not periodic.



          So for a positive integer $m$, the function is periodic iff $m$ is odd, no matter what positive integer $a_1$ is.



          [If $m$ is 0 and $a_1 $ is odd then all of the $a_n$s are $a_1$. If $m=0$ and $a_1$ is even then letting $i_0$ be the smallest integer s.t. $a_12^{-i_0}$ is odd, then $a_n=a_12^{-i_0}$ for all $n > i_0$.]





          Proof that $a_k< a_1+3m$ for all $k$ if $m$ is odd: We first show that $a_k < a_1+2m$ if $a_k$ is odd, by induction on $k$. Let us assume that $a_{k_0} < a_1+2m$ and that $a_{k_0}$ is odd. Then $a_{k_0+1}=a_{k_0}+m$ is even [as $m$ is odd] and is no greater than $a_1+3m$. Then $a_{k_0+2} = a_{k_0+1}/2$ is no greater than $a_1/2 + 3m/2 < a_1+2m$. So indeed, $a_k < a_1+2m$ if $a_k$ is odd.



          Now we claim that $a_k < a_1+3m$ if $a_k$ is even. Indeed, let us use induction on $k$. If $a_{k-1}$ is even then by induction hypthesis $a_{k-1} < a_1+3m$ and $a_k = a_{k-1}/2 < a_{k-1}$ so indeed, $a_k < a_1+3m$ as well. If $a_{k-1}$ is odd then by the previous paragraph $a_{k-1} < a_1+2m$, which implies $a_k < a_1+3m$. Thus indeed, $a_k < a_1+3m$ if $a_k$ is even.



          So $a_k < a_1+2m$ if $a_k$ is odd and $a_k < a_1+3m$ if $a_k$ is even, so the $a_k$s are indeed bounded.






          share|cite|improve this answer











          $endgroup$














            Your Answer








            StackExchange.ready(function() {
            var channelOptions = {
            tags: "".split(" "),
            id: "69"
            };
            initTagRenderer("".split(" "), "".split(" "), channelOptions);

            StackExchange.using("externalEditor", function() {
            // Have to fire editor after snippets, if snippets enabled
            if (StackExchange.settings.snippets.snippetsEnabled) {
            StackExchange.using("snippets", function() {
            createEditor();
            });
            }
            else {
            createEditor();
            }
            });

            function createEditor() {
            StackExchange.prepareEditor({
            heartbeatType: 'answer',
            autoActivateHeartbeat: false,
            convertImagesToLinks: true,
            noModals: true,
            showLowRepImageUploadWarning: true,
            reputationToPostImages: 10,
            bindNavPrevention: true,
            postfix: "",
            imageUploader: {
            brandingHtml: "Powered by u003ca class="icon-imgur-white" href="https://imgur.com/"u003eu003c/au003e",
            contentPolicyHtml: "User contributions licensed under u003ca href="https://creativecommons.org/licenses/by-sa/3.0/"u003ecc by-sa 3.0 with attribution requiredu003c/au003e u003ca href="https://stackoverflow.com/legal/content-policy"u003e(content policy)u003c/au003e",
            allowUrls: true
            },
            noCode: true, onDemand: true,
            discardSelector: ".discard-answer"
            ,immediatelyShowMarkdownHelp:true
            });


            }
            });














            draft saved

            draft discarded


















            StackExchange.ready(
            function () {
            StackExchange.openid.initPostLogin('.new-post-login', 'https%3a%2f%2fmath.stackexchange.com%2fquestions%2f3071102%2ffind-values-of-a-for-which-the-function-is-periodic%23new-answer', 'question_page');
            }
            );

            Post as a guest















            Required, but never shown

























            1 Answer
            1






            active

            oldest

            votes








            1 Answer
            1






            active

            oldest

            votes









            active

            oldest

            votes






            active

            oldest

            votes









            0












            $begingroup$


            Fact 1: If there are two distinct $k_1$ and $k_2$ s.t. $a_{k_1}=a_{k_2}$ then the sequence repeats.




            If $m$ is odd (the crux of the problem) then each $a_k$ is no larger than $a_1 + 3m$ [make sure you see why or check the proof below; essentially you get to add by $m$ only once before you have to divide by 2]. As each $a_k$ is positive it follows that $a_k in {1,2,ldots, a_1+3m}$ for all $k$. Therefore it follows from Fact 1 that the sequence repeats.



            If $m$ is even and positive then let $i_0$ be the smallest integer such that $a_12^{-i_0}$ is no longer even--or equivalently $2^{-i_0}a_1$ is odd. Then $a_2 = a_12^{-1}$, $ldots ,$ $a_{i_0+1} = 2^{-i_0}a_1$, while $a_{i_0+1+j} = a_12^{-i_0} + jm$ for each nonnegative integer $j$ [make sure you see why; note that $a_{i_0+j+1} = a_12^{-i_0} + jm$ is an odd integer as $a_12^{-i_0}$ is odd and $m$ is even]. So the function is not periodic.



            So for a positive integer $m$, the function is periodic iff $m$ is odd, no matter what positive integer $a_1$ is.



            [If $m$ is 0 and $a_1 $ is odd then all of the $a_n$s are $a_1$. If $m=0$ and $a_1$ is even then letting $i_0$ be the smallest integer s.t. $a_12^{-i_0}$ is odd, then $a_n=a_12^{-i_0}$ for all $n > i_0$.]





            Proof that $a_k< a_1+3m$ for all $k$ if $m$ is odd: We first show that $a_k < a_1+2m$ if $a_k$ is odd, by induction on $k$. Let us assume that $a_{k_0} < a_1+2m$ and that $a_{k_0}$ is odd. Then $a_{k_0+1}=a_{k_0}+m$ is even [as $m$ is odd] and is no greater than $a_1+3m$. Then $a_{k_0+2} = a_{k_0+1}/2$ is no greater than $a_1/2 + 3m/2 < a_1+2m$. So indeed, $a_k < a_1+2m$ if $a_k$ is odd.



            Now we claim that $a_k < a_1+3m$ if $a_k$ is even. Indeed, let us use induction on $k$. If $a_{k-1}$ is even then by induction hypthesis $a_{k-1} < a_1+3m$ and $a_k = a_{k-1}/2 < a_{k-1}$ so indeed, $a_k < a_1+3m$ as well. If $a_{k-1}$ is odd then by the previous paragraph $a_{k-1} < a_1+2m$, which implies $a_k < a_1+3m$. Thus indeed, $a_k < a_1+3m$ if $a_k$ is even.



            So $a_k < a_1+2m$ if $a_k$ is odd and $a_k < a_1+3m$ if $a_k$ is even, so the $a_k$s are indeed bounded.






            share|cite|improve this answer











            $endgroup$


















              0












              $begingroup$


              Fact 1: If there are two distinct $k_1$ and $k_2$ s.t. $a_{k_1}=a_{k_2}$ then the sequence repeats.




              If $m$ is odd (the crux of the problem) then each $a_k$ is no larger than $a_1 + 3m$ [make sure you see why or check the proof below; essentially you get to add by $m$ only once before you have to divide by 2]. As each $a_k$ is positive it follows that $a_k in {1,2,ldots, a_1+3m}$ for all $k$. Therefore it follows from Fact 1 that the sequence repeats.



              If $m$ is even and positive then let $i_0$ be the smallest integer such that $a_12^{-i_0}$ is no longer even--or equivalently $2^{-i_0}a_1$ is odd. Then $a_2 = a_12^{-1}$, $ldots ,$ $a_{i_0+1} = 2^{-i_0}a_1$, while $a_{i_0+1+j} = a_12^{-i_0} + jm$ for each nonnegative integer $j$ [make sure you see why; note that $a_{i_0+j+1} = a_12^{-i_0} + jm$ is an odd integer as $a_12^{-i_0}$ is odd and $m$ is even]. So the function is not periodic.



              So for a positive integer $m$, the function is periodic iff $m$ is odd, no matter what positive integer $a_1$ is.



              [If $m$ is 0 and $a_1 $ is odd then all of the $a_n$s are $a_1$. If $m=0$ and $a_1$ is even then letting $i_0$ be the smallest integer s.t. $a_12^{-i_0}$ is odd, then $a_n=a_12^{-i_0}$ for all $n > i_0$.]





              Proof that $a_k< a_1+3m$ for all $k$ if $m$ is odd: We first show that $a_k < a_1+2m$ if $a_k$ is odd, by induction on $k$. Let us assume that $a_{k_0} < a_1+2m$ and that $a_{k_0}$ is odd. Then $a_{k_0+1}=a_{k_0}+m$ is even [as $m$ is odd] and is no greater than $a_1+3m$. Then $a_{k_0+2} = a_{k_0+1}/2$ is no greater than $a_1/2 + 3m/2 < a_1+2m$. So indeed, $a_k < a_1+2m$ if $a_k$ is odd.



              Now we claim that $a_k < a_1+3m$ if $a_k$ is even. Indeed, let us use induction on $k$. If $a_{k-1}$ is even then by induction hypthesis $a_{k-1} < a_1+3m$ and $a_k = a_{k-1}/2 < a_{k-1}$ so indeed, $a_k < a_1+3m$ as well. If $a_{k-1}$ is odd then by the previous paragraph $a_{k-1} < a_1+2m$, which implies $a_k < a_1+3m$. Thus indeed, $a_k < a_1+3m$ if $a_k$ is even.



              So $a_k < a_1+2m$ if $a_k$ is odd and $a_k < a_1+3m$ if $a_k$ is even, so the $a_k$s are indeed bounded.






              share|cite|improve this answer











              $endgroup$
















                0












                0








                0





                $begingroup$


                Fact 1: If there are two distinct $k_1$ and $k_2$ s.t. $a_{k_1}=a_{k_2}$ then the sequence repeats.




                If $m$ is odd (the crux of the problem) then each $a_k$ is no larger than $a_1 + 3m$ [make sure you see why or check the proof below; essentially you get to add by $m$ only once before you have to divide by 2]. As each $a_k$ is positive it follows that $a_k in {1,2,ldots, a_1+3m}$ for all $k$. Therefore it follows from Fact 1 that the sequence repeats.



                If $m$ is even and positive then let $i_0$ be the smallest integer such that $a_12^{-i_0}$ is no longer even--or equivalently $2^{-i_0}a_1$ is odd. Then $a_2 = a_12^{-1}$, $ldots ,$ $a_{i_0+1} = 2^{-i_0}a_1$, while $a_{i_0+1+j} = a_12^{-i_0} + jm$ for each nonnegative integer $j$ [make sure you see why; note that $a_{i_0+j+1} = a_12^{-i_0} + jm$ is an odd integer as $a_12^{-i_0}$ is odd and $m$ is even]. So the function is not periodic.



                So for a positive integer $m$, the function is periodic iff $m$ is odd, no matter what positive integer $a_1$ is.



                [If $m$ is 0 and $a_1 $ is odd then all of the $a_n$s are $a_1$. If $m=0$ and $a_1$ is even then letting $i_0$ be the smallest integer s.t. $a_12^{-i_0}$ is odd, then $a_n=a_12^{-i_0}$ for all $n > i_0$.]





                Proof that $a_k< a_1+3m$ for all $k$ if $m$ is odd: We first show that $a_k < a_1+2m$ if $a_k$ is odd, by induction on $k$. Let us assume that $a_{k_0} < a_1+2m$ and that $a_{k_0}$ is odd. Then $a_{k_0+1}=a_{k_0}+m$ is even [as $m$ is odd] and is no greater than $a_1+3m$. Then $a_{k_0+2} = a_{k_0+1}/2$ is no greater than $a_1/2 + 3m/2 < a_1+2m$. So indeed, $a_k < a_1+2m$ if $a_k$ is odd.



                Now we claim that $a_k < a_1+3m$ if $a_k$ is even. Indeed, let us use induction on $k$. If $a_{k-1}$ is even then by induction hypthesis $a_{k-1} < a_1+3m$ and $a_k = a_{k-1}/2 < a_{k-1}$ so indeed, $a_k < a_1+3m$ as well. If $a_{k-1}$ is odd then by the previous paragraph $a_{k-1} < a_1+2m$, which implies $a_k < a_1+3m$. Thus indeed, $a_k < a_1+3m$ if $a_k$ is even.



                So $a_k < a_1+2m$ if $a_k$ is odd and $a_k < a_1+3m$ if $a_k$ is even, so the $a_k$s are indeed bounded.






                share|cite|improve this answer











                $endgroup$




                Fact 1: If there are two distinct $k_1$ and $k_2$ s.t. $a_{k_1}=a_{k_2}$ then the sequence repeats.




                If $m$ is odd (the crux of the problem) then each $a_k$ is no larger than $a_1 + 3m$ [make sure you see why or check the proof below; essentially you get to add by $m$ only once before you have to divide by 2]. As each $a_k$ is positive it follows that $a_k in {1,2,ldots, a_1+3m}$ for all $k$. Therefore it follows from Fact 1 that the sequence repeats.



                If $m$ is even and positive then let $i_0$ be the smallest integer such that $a_12^{-i_0}$ is no longer even--or equivalently $2^{-i_0}a_1$ is odd. Then $a_2 = a_12^{-1}$, $ldots ,$ $a_{i_0+1} = 2^{-i_0}a_1$, while $a_{i_0+1+j} = a_12^{-i_0} + jm$ for each nonnegative integer $j$ [make sure you see why; note that $a_{i_0+j+1} = a_12^{-i_0} + jm$ is an odd integer as $a_12^{-i_0}$ is odd and $m$ is even]. So the function is not periodic.



                So for a positive integer $m$, the function is periodic iff $m$ is odd, no matter what positive integer $a_1$ is.



                [If $m$ is 0 and $a_1 $ is odd then all of the $a_n$s are $a_1$. If $m=0$ and $a_1$ is even then letting $i_0$ be the smallest integer s.t. $a_12^{-i_0}$ is odd, then $a_n=a_12^{-i_0}$ for all $n > i_0$.]





                Proof that $a_k< a_1+3m$ for all $k$ if $m$ is odd: We first show that $a_k < a_1+2m$ if $a_k$ is odd, by induction on $k$. Let us assume that $a_{k_0} < a_1+2m$ and that $a_{k_0}$ is odd. Then $a_{k_0+1}=a_{k_0}+m$ is even [as $m$ is odd] and is no greater than $a_1+3m$. Then $a_{k_0+2} = a_{k_0+1}/2$ is no greater than $a_1/2 + 3m/2 < a_1+2m$. So indeed, $a_k < a_1+2m$ if $a_k$ is odd.



                Now we claim that $a_k < a_1+3m$ if $a_k$ is even. Indeed, let us use induction on $k$. If $a_{k-1}$ is even then by induction hypthesis $a_{k-1} < a_1+3m$ and $a_k = a_{k-1}/2 < a_{k-1}$ so indeed, $a_k < a_1+3m$ as well. If $a_{k-1}$ is odd then by the previous paragraph $a_{k-1} < a_1+2m$, which implies $a_k < a_1+3m$. Thus indeed, $a_k < a_1+3m$ if $a_k$ is even.



                So $a_k < a_1+2m$ if $a_k$ is odd and $a_k < a_1+3m$ if $a_k$ is even, so the $a_k$s are indeed bounded.







                share|cite|improve this answer














                share|cite|improve this answer



                share|cite|improve this answer








                edited Jan 12 at 23:21

























                answered Jan 12 at 17:00









                MikeMike

                4,641512




                4,641512






























                    draft saved

                    draft discarded




















































                    Thanks for contributing an answer to Mathematics Stack Exchange!


                    • Please be sure to answer the question. Provide details and share your research!

                    But avoid



                    • Asking for help, clarification, or responding to other answers.

                    • Making statements based on opinion; back them up with references or personal experience.


                    Use MathJax to format equations. MathJax reference.


                    To learn more, see our tips on writing great answers.




                    draft saved


                    draft discarded














                    StackExchange.ready(
                    function () {
                    StackExchange.openid.initPostLogin('.new-post-login', 'https%3a%2f%2fmath.stackexchange.com%2fquestions%2f3071102%2ffind-values-of-a-for-which-the-function-is-periodic%23new-answer', 'question_page');
                    }
                    );

                    Post as a guest















                    Required, but never shown





















































                    Required, but never shown














                    Required, but never shown












                    Required, but never shown







                    Required, but never shown

































                    Required, but never shown














                    Required, but never shown












                    Required, but never shown







                    Required, but never shown







                    Popular posts from this blog

                    Cabo Verde

                    Karlovacs län

                    Gyllenstierna